Forme exponentielle complexe

Bonjour Bonjour, je viens vers vous car je rencontre un problème (qui est certainement tout bête comme d'habitude... mais je bute).

Je dois déterminer le module et l'argument du nombre complexe suivant : $$e^{e^{i\theta}},\quad \theta \in \mathbb{R}
$$ Bon je sais qu'un nombre complexe est de la forme $z = a + ib$
Je sais également que la forme trigonométrique d'un nombre complexe est par définition : $|z|\big( \cos(\theta) + i\sin(\theta) \big)$
Je sais aussi que la forme exponentielle d'un nombre complexe est : $|z|e^{i\theta}$.
Là j'ai donc : $$e^{e^{i\theta}},\quad \theta \in \mathbb{R}
$$ Au début je me suis dit que la forme trigonométrique était : $\cos(e^{\theta}) + i\sin(e^{\theta})$
Mais je me suis vite dit que c'était idiot...
Pourriez-vous me mettre sur une piste ? (J'essaie de trouver la forme algébrique pour commencer, ensuite je calculerais le module puis l'argument. Le module semble être 1).

Merci !

Réponses

  • Bonjour,

    Écris l’argument de l’exponentielle sous forme algébrique...
  • Bonjour, merci de votre réponse.

    Je pensais tout d'abord trouver la forme algébrique de ce nombre complexe.. afin d'en calculer le module (qui semble être ici 1) et en trouver l'argument ?

    Par ailleurs, je ne comprend pas spécialement votre message, écrire un argument sous forme algébrique ? je sais que $arg(z) = \theta$.
  • Il te suffit d'écrire $\mathrm{e}^{i \theta}$ sous forme algébrique (trigonométrique si tu veux), puis de passer à l'exponentielle !
  • Je vois, de ce fait : $$e^{e^{i\theta}} = e^{cos(\theta) + isin(\theta)}$$ ?
  • Oui, et maintenant il te suffit d'identifier le module et l'argument.
  • Alors je pense avoir trouvé : $$e^{e^{i\theta}} = e^{\cos(\theta) + i\sin(\theta)} = e^{\cos(\theta)} \times e^{i\sin(\theta)}
    $$ D'où : $$|z| = e^{\cos(\theta)} \quad\text{et}\quad arg(z) = \sin(\theta)
    $$ Qu'en pensez-vous ?
  • J'en pense que tu n'as jamais défini ce qu'est $z$ donc je ne peux pas juger (:P)

    Plus sérieusement, qu'en penses-tu toi ? Il faut que tu sois sûr de ce que tu écris.
  • Ah oui pardon !!!
    Quel idiot ! $\quad z = e^{e^{i\theta}}$
    Je suis totalement convaincu de ma démarche et de mon résultat !
  • Et pourrais-tu argumenter ton identification $|z| = \exp(\cos(\theta))$ et $\arg(z) = \sin(\theta)$ ?
  • Oui bien sûr, je vais vous rédiger ce que j'ai écris sur mon brouillon.

    On pose : $z = e^{e^{i\theta}}$

    $$\exists \alpha \in \mathbb{C} | \alpha = e^{i\theta}$$

    Soit : $e^{\alpha} = e^{e^{i\theta}}$

    $f_{trigonométrique \rightarrow \alpha} = cos(\theta) + isin(\theta).$

    Donc :

    $e^{\alpha} = e^{cos(\theta) + isin(\theta)}$
    $e^{\alpha} = e^{cos(\theta)} \times e^{isin(\theta)}$.

    Nous reconnaissons la forme $|z|e^{i\theta}$.

    Conclusion :

    $$|z| = e^{cos(\theta)}$$ et $$arg(z) = sin(\theta)$$
  • Il te manque essentiellement deux arguments pour conclure. Si je te donne le nombre $- \sqrt 2 \mathrm{e}^{i \mathrm{e}^{2 \pi}}$, comment identifies-tu le module et l'argument ?
  • Je dirais que l'argument ici c'est $e^{2\pi}[\pi]$
    et que le module est : $-\sqrt{2}$.

    Je l'identifie car je sais connais l'exponentielle complexe non ?


    Je vois pas vraiment où vous voulez en venir.. ?

    Mais j'aimerai comprendre ! :)
  • Penses-tu vraiment qu'un nombre complexe peut avoir pour module $- \sqrt 2$ ? Que représente le module géométriquement ?
  • Oui effectivement, je suis allé trop vite un module ne peut pas être négatif... Je ne vois vraiment pas..

    Un peu d'éclairage serait le bienvenue !
  • Il suffit d'appliquer à la lettre (et autant de fois que nécessaire) la relation bien connue : $e^{i\theta}= \cos\theta+i\sin\theta$
    $e^{e^{i\theta}}= e^{\cos\theta+i\sin\theta}= e^{\cos\theta}e^{i\sin\theta}$
    et $e^{i\sin\theta}= ...$
  • Lucas13,

    $r e^{it}$ est la forme exponentielle d'un complexe si r>0 (je laisse de côté le cas de 0). Et dans ce cas, r est son module.
    Si r est négatif, on sait l'écrire sous forme exponentielle pour retomber sur la bonne écriture.
    Dans ton cas, il ne manquait que de remarquer que $e^{\sin\theta}$ est positif.

    Cordialement.

    [édit : corrigé grâce à Poirot : $e^{\sin\theta}$ à la place de $e^{i\sin\theta}$ copié-collé trop vite]
  • Bonjour @Luca13,

    Soit $e^{e^{i \theta}},\ \theta \in \R.$

    Ce truc est un nombre complexe. On a $e^{i \theta} = \cos \theta + i \sin \theta,\ \theta \in \R$ : c'est la formule d'Euler. On a donc : $e^{e^{i \theta}} = e^{ \cos \theta + i \sin \theta},\ \theta \in \R.$ On sait les propriétés de la fonction exponentielle et donc on a aussi $e^{e^{i \theta}} = e^{ \cos \theta }e^{ i \sin \theta},\ \theta \in \R.$

    Pour identifier le module, il faut remarquer que $e^{ \cos \theta }>0,\ \theta \in \R$ et $|e^{ i \sin \theta}| = 1,\ \theta \in \R.$
    Pour l'argument, c'est plus compliqué car on peut écrire $e^{ i \sin \theta} = e^{ i \sin \theta + i 2 \pi m},\ m \in \Z,\ \theta \in \R.$ Et donc $arg (e^{ i \sin \theta} )= \sin \theta + 2 \pi m,\ m \in \Z.$

    Pour appliquer tout ça, un petit exercice : quel est le module et quel est l'argument de ce nombre complexe : $\cos (x) e^{-i x},\ x \in \R$ ?
  • @gerard : voulais-tu vraiment écrire "$\mathrm{e}^{i \sin \theta}$ est positif" ?

    Bon sinon YvesM t'a tout donné, il aurait pu te laisser chercher seul...
  • Bonjour Yves, merci à vous je viens d'étudier de très près votre proposition effectivement c'est ce qu'il fallait. Je vais faire votre petit exo et je vous tiens au courant. Néanmoins je n'ai pas saisi pourquoi $|e^{i\sin(\theta)}| = 1$ et du moins pourquoi avons-nous besoin de faire ceci ?
    Je vous remercie !
  • Merci Poirot, je rectifie.
  • Si tu as $z=ab$ avec $|b|=1$ alors tu as presque la forme exponentiel de $z$ : $z=|a| \mathrm{e}^{i \arg(a) + i \theta}$ où $\theta$ est tel que $b= \mathrm{e}^{i \theta}$. Ici le $a$ est $\mathrm{e}^{\cos \theta}$ est un nombre réel strictement positif donc est déjà sous sa forme exponentielle.
  • D'accord, oui je vois merci Poirot, mais je ne vois pas pourquoi |b| = 1 enfin je ne vois pas surtout qu'est ce qui me fait dire ça vous voyez ?
  • é je ne vois pas pourquoi |b| = 1" ??

    Tu ne sais pas que si t est un réel, $e^{it}$ est de module 1 ? C'est pourtant dans quasiment tous les cours sur la forme exponentielle, et se démontre immédiatement avec la définition, non ?

    Tu ferais bien de relire sérieusement tes cours pour savoir ce qui est "évident".

    Cordialement.
  • Bonjour Gerard0

    Si bien sûr que je sais que $e^{it} = \cos t + i\sin t$ correspond à la notation exponentielle d'un nombre complexe dont le module vaut 1.

    Mais ce que je ne saisi pas c'est pourquoi $|e^{ i\sin \theta}|$ $= 1$. Enfin si le module est 1 car il est de la forme $e^{it}$ mais pourquoi le mentionner dans ma résolution ?

    Merci !
  • Ben ... $\sin(\theta)$ est bien un réel !

    Sinon, pour en revenir à ta rédaction, ce qui manquait n'est pas que $|e^{i\sin(\theta)}|=1$, mais que $e^{\cos(\theta)}>0$.
    Cours : voir ce message.

    Face à tes incompréhensions, certains sont revenus à des éléments de preuve de cette propriété classique qu'on voit en cours.

    Cordialement.

    NB : Quand on connaît mal ses leçons, on pose des questions qui donnent l'idée qu'on ne comprend pas de quoi on parle. Ici, tu l'as fait plusieurs fois. Quand on les connaît vraiment, on n'a même plus besoin de l'avis des autres, on sait ce qui est correct.
  • Je viens de relire mon cours et il n'est pas explicité que $e^{i\theta}$ à toujours pour module 1 (je suis en L1 mathématiques).

    Le Professeur nous montre juste un exemple avec zz' où z et z' ont pour module 1.
  • Peut-être que $\cos^2+\sin^2=1$ peut servir un peu ?
  • Bonjour ModuloP, effectivement suis-je bête... j'arrive pas acquérir cet automatisme ! Il faut que j'accentue mes efforts !
  • Pour les automatismes, la solution est très simple : travailler ;)
    Bon courage,
  • Encore plus simple si tu sais que $\overline{\mathrm{e}^{i \theta}} = \mathrm{e}^{-i \theta}$. Alors $\left|\mathrm{e}^{i \theta}\right|^2 = \mathrm{e}^{i \theta} \mathrm{e}^{-i \theta} = \mathrm{e}^0 = 1$.
Connectez-vous ou Inscrivez-vous pour répondre.